PDA

View Full Version : Why does FTL violate causality?



keybounce
2018-11-14, 11:47 AM
In the science fiction vs fantasy thread, on the subject of hard sci-fi vs soft sci-fi, I constantly see people complain that FTL violates causality and is equal to time travel.

And: Why?

Lets say I go from Sol to Alpha Centauri in less than 5 minutes. I gather information about the sun, and return back home. I then say "this is what will happen 4 years from now".

But, I still returned to Sol about 10 minutes after I left.

I've seen people point out that if your FTL travel uses your frame of reference -- specifically, the frame of reference you have before you go FTL -- then two round trips can, by changing reference frame at the stop-over -- generate a time loop.

But why does FTL have to use the originator's frame of reference? Why wouldn't it use it's own FoR that is based on the FTL medium / whatever you travel through to make the trip?

And once you have a single constant FoR for all trips, doesn't that make time travel by FTL travel impossible?

Yora
2018-11-14, 11:56 AM
As science goes, anything going faster than light is magic.

That being said, hypothetical faster than light movement only "violates causality" in the sense that light travel at a speed equal to the speed of causality. If you move faster than c, you violate c.
But even then, it does not cause an effect to happen before its own cause.

Anonymouswizard
2018-11-14, 12:03 PM
The short answer is 'there is no authoritive frame of reference to gauge FTL travel from'.

The slightly longer and maybe somewhat wrong (I have never actually worked with physics at this level) is that we have the origin's frame, the traveller's frame, and the destination frame, as well as a bunch of other frames because other things in the universe exist. But essentially for causality to work you need events to play out (or be detected) in the same order in each frame of reference. FTL and some fancy movement can allow you to have events in one frame play out in a different order in another frame.

A friend once tried to explain to me why this happens using the fact that once you hit relativistic speeds sizes appear to change, and you can use theoretically use this (this might have required actual FTL travel, it's been years) to get a plank in a shed half it's size, close both doors, than open the exit door and let it out because of how even here we have to disagree on the order of events, from our frame of reference we had both doors closed, while from the plank's we had to open the exit door before we could close the entrance because otherwise the plank would never fit in.

Also this might be better for the 'Mad Science and Grumpy Technology' section. This thread (http://www.giantitp.com/forums/showthread.php?548530-FTL-and-Violating-Causality&highlight=FTL+causality) from there might be good reading material. In essence, you can construct an FTL system that avoids time travel, but you need to destroy Relativity to pull it off.

Ibrinar
2018-11-14, 12:13 PM
There is to the best of our knowledge no preferred reference frame, but physics is the same in all reference frame. You could of course introduce one in scific but methods to explain why it doesn't work in your setting is another topic. If you can travel ftl in all frames, why it breaks causality is probably easiest to understand with instant transmission (it works with just ftl too but you actually have to do the math for the timings then.)

One important thing about relativity is that events simultaneous in one reference frame can happen in a different order in a different reference frame. (https://en.wikipedia.org/wiki/Relativity_of_simultaneity) If there is an event T and event M all frames will only agree T happened after M if light of T could have reached M, otherwise some will think M happened first. (https://en.wikipedia.org/wiki/Light_cone) (if I didn't get that wrong.)

Take two planets A and B who are far away from each other but move in the same way so no speed relative to each other. Event X happens on planet A, they sent an instant message to planet B, planet B reacts by immediately informing Observer C with instant transmission. Now the observer C moves with a high fraction of C in relation to the two and in such a way that B receiving the message happens from their perspective before A sends the message (as mentioned simultaneous event aren't necessarily simultaneous from another frame) so from their perspective they received the message before Event X happened. If they now use instant transmission to alert A they can alert them of X before X happened.

Willie the Duck
2018-11-14, 12:26 PM
Lets say I go from Sol to Alpha Centauri in less than 5 minutes. I gather information about the sun, and return back home. I then say "this is what will happen 4 years from now".

But, I still returned to Sol about 10 minutes after I left.

This, by itself, is why it violated causality. If you can go find out what happens 4 years from now, and go back and change what will happen, you have violated causality. That's the primary issue. While we don't know that FTL is straight up impossible (so much as we haven't found a way to do it), we really don't know that time travel violates the laws of physics, only that if it were possible, one could violate the laws of logic.

Reddish Mage
2018-11-14, 12:40 PM
Shouldn't this be in the "Mad Science" forum?

Narkis
2018-11-14, 01:00 PM
There is to the best of our knowledge no preferred reference frame, but physics is the same in all reference frame. You could of course introduce one in scific but methods to explain why it doesn't work in your setting is another topic. If you can travel ftl in all frames, why it breaks causality is probably easiest to understand with instant transmission (it works with just ftl too but you actually have to do the math for the timings then.)

One important thing about relativity is that events simultaneous in one reference frame can happen in a different order in a different reference frame. (https://en.wikipedia.org/wiki/Relativity_of_simultaneity) If there is an event T and event M all frames will only agree T happened after M if light of T could have reached M, otherwise some will think M happened first. (https://en.wikipedia.org/wiki/Light_cone) (if I didn't get that wrong.)

Take two planets A and B who are far away from each other but move in the same way so no speed relative to each other. Event X happens on planet A, they sent an instant message to planet B, planet B reacts by immediately informing Observer C with instant transmission. Now the observer C moves with a high fraction of C in relation to the two and in such a way that B receiving the message happens from their perspective before A sends the message (as mentioned simultaneous event aren't necessarily simultaneous from another frame) so from their perspective they received the message before Event X happened. If they now use instant transmission to alert A they can alert them of X before X happened.

I don't understand how this can happen as stated. Yes, C will get the message from B before they see it being sent from A, but that should have no relevance on A. From A's point of view, something happened, THEN they immediately notified B, THEN they got immediately notified by C. No causality violations whatsoever. C might be a bit confused about the order of events, but it should not be too hard to clear things out.


This, by itself, is why it violated causality. If you can go find out what happens 4 years from now, and go back and change what will happen, you have violated causality. That's the primary issue. While we don't know that FTL is straight up impossible (so much as we haven't found a way to do it), we really don't know that time travel violates the laws of physics, only that if it were possible, one could violate the laws of logic.

But you don't go back in time. You notify Earth about what Earth will SEE happening in 4 years. The event has still already happened. Even if you immediately head back to Alpha Centauri, you will get there 10 minutes after the event, unable to change a thing.

DavidSh
2018-11-14, 01:06 PM
Shouldn't this be in the "Mad Science" forum?
It was, at one point, with the usual suspects complaining that a stronger version of the claim (that nobody ever made) was obvious nonsense.

The short answer is that, if Special Relativity is true, and there are no preferred inertial reference frames, and you can make an FTL move with distance X and time T, (so X/T > speed of light) with respect to one inertial frame, then you can make an FTL move with distance X and time T with respect to any inertial frame. Otherwise there would be a subset of preferred inertial reference frames. If you can string together such FTL moves with respect to multiple arbitrary inertial frames, you can travel from a point (x,t) in spacetime to a point (x,t') which is in the past with respect to (x,t) (but at the same location with respect to some inertial reference frame), allowing for a loop in time.


Furthermore, any FTL move is travel into the past with respect to some set of inertial frames, but not every sequence of FTL moves necessarily creates loops.

A more exact analysis gets into light cones, and time-like and space-like intervals.

Z3ro
2018-11-14, 01:08 PM
You're making a simple mistake; as you go faster, time slows down. So as you approach then break the speed of light, weird things happen to the time axis with regards to your movement. The truth is we don't know what would happen, as we've never been able to observe anything going FTL.

In your example of going to Alpha Centauri, if you were able to do so in a way that didn't involve FTL travel (say, a wormhole or an Alcubierre warp drive, you would in fact be able to do the thing you describe, and it wouldn't be time travel. It's the FTL that's the problem.

LibraryOgre
2018-11-14, 01:11 PM
This, by itself, is why it violated causality. If you can go find out what happens 4 years from now, and go back and change what will happen, you have violated causality. That's the primary issue. While we don't know that FTL is straight up impossible (so much as we haven't found a way to do it), we really don't know that time travel violates the laws of physics, only that if it were possible, one could violate the laws of logic.

But I wouldn't say you can find out what will happen in 4 years... you can find out what will be visible in 4 years, from a given point.

Let's say I go FTL, blow up Alpha Centauri, and return. In 4 years, my blowing up Alpha Centauri will be visible from Earth, because light from the Alpha Centauri Nova will take that long to reach Earth. But that doesn't mean you can go back in time to stop me, unless you can actually go back in time. The INFORMATION about the event is taking time to travel... the event itself has already happened.

I find the other way of looking at it like saying you can stop someone from hitting a wood block after you've seen it, but before the sound reaches you. You can stop a bullet from being fired if you see the muzzle flash then travel faster than sound to the sniper. It seems ridiculous to say, because we accept that the bullet has already left when we see the muzzle flash (which traveled faster than sound).


Shouldn't this be in the "Mad Science" forum?

I was thinking the same thing, but figure it's barely acceptable because of the media impact of it. But I'm flexible on it.

Saph
2018-11-14, 01:25 PM
I'm not a good enough physics teacher to explain it very well, but the simple version that my friend (who is a physics teacher) has told me is that you get to pick two out of three:

1) Relativity theory
2) Faster-than-light travel
3) Causality

Dropping causality is generally considered the worst option for obvious reasons. Dropping relativity might seem appealing at first glance, but the problem is, it's been confirmed again and again over decades of work via every test that scientists have been able to devise (in fact, it's a necessary component of some of the technology you use).

So if one of the three is going to get the axe, there's a real obvious candidate.

Ibrinar
2018-11-14, 01:32 PM
I don't understand how this can happen as stated. Yes, C will get the message from B before they see it being sent from A, but that should have no relevance on A. From A's point of view, something happened, THEN they immediately notified B, THEN they got immediately notified by C. No causality violations whatsoever. C might be a bit confused about the order of events, but it should not be too hard to clear things out.



If in Cs frame of reference instant transmission is possible that means that their message arrives at A in what they consider to be A s present which is before event X. Since the message arriving and event X happen in about the same location, event X is in the light cone of the message arriving and thus everyone agrees that event X happened after the message about X arriving at A.

Edit: C is not confused, order of events is frame dependent unless event A is in the light cone of event B. With any non ftl speed you can't influence anything not in your light cone, with ftl you can. The message could arrive minutes/hours prior to X. If that isn't so it in't instant in one of the frames. If A only gets the message when they send theirs then in Cs frame their reaction to Bs message is not instant.

Edit 2: The explanation Grey_Wolf_c links below is more understandable than mine I think.

Aotrs Commander
2018-11-14, 01:40 PM
But I wouldn't say you can find out what will happen in 4 years... you can find out what will be visible in 4 years, from a given point.

Let's say I go FTL, blow up Alpha Centauri, and return. In 4 years, my blowing up Alpha Centauri will be visible from Earth, because light from the Alpha Centauri Nova will take that long to reach Earth. But that doesn't mean you can go back in time to stop me, unless you can actually go back in time. The INFORMATION about the event is taking time to travel... the event itself has already happened.

I find the other way of looking at it like saying you can stop someone from hitting a wood block after you've seen it, but before the sound reaches you. You can stop a bullet from being fired if you see the muzzle flash then travel faster than sound to the sniper. It seems ridiculous to say, because we accept that the bullet has already left when we see the muzzle flash (which traveled faster than sound).

Or, as an example which is basically the same thing on a smaller scale, if I just cruise out at, let's say, cruisng speed of the Aotrs starfleet on sublight engines (a touch over 10% lightspeed) which will take me about 80 minutes) and blow up the sun, it will take eight minutes for anyone to realise it on Earth - and about 80 minutes (based on the speed of a supernova, which is, as it happens about 10% c as well1) until the shockwave hit.

No time travel is involved.

It's the whole "requires infinite energy to reach the speed of light" which is part of the problem and that the speed of light is the maximum speed of transfer of information.

FTL doesn't really have anything to do with time-travel per se, but as I understand it, (theorhetical) uncompenstated c-transit messes with the local time of the traveller (in very broad, non-technical terms).



(Compenstated c-transit is part of how Aotrs railguns can fire at 2-3 c, using the same temporal compensation as the engines. It's way out of my league, (in fact I got it so badly wrong once, I had to get of my weapons tech tom explain and even then I was mostly lost), but the lies-to-children part of it is that increasing the effective velocity beyond c doesn't actually increase the damage (because of course, if you can accelerate to c without compensation you have infinite power or whatever), but essentially mostly just drops the transit speed - which when you are firing at ranges of around a third of a lightsecond at a target moving at something between 10-60% c makes a huge difference. But that's a whole other complicated issue that, as I say, I very poorly understand, since I'm a necromancer, not a weapons tech...)





1So, disregarding Aotrs Gate drive (which is somewhere between a wormhole and teleportation, but let's not get into that), we'd have to book it back, but the distance is enough we could get up to max safe sublight speed (roughly 60% c) to have time to gloat...

Gallowglass
2018-11-14, 01:58 PM
This, by itself, is why it violated causality. If you can go find out what happens 4 years from now, and go back and change what will happen, you have violated causality. That's the primary issue. While we don't know that FTL is straight up impossible (so much as we haven't found a way to do it), we really don't know that time travel violates the laws of physics, only that if it were possible, one could violate the laws of logic.

Really, to me, this seems like a semantics issue.

Assuming you had the ability to move from Sol to Alpha Centauri in (the frame of reference you call five minutes) then back in five more minutes, arriving 10 minutes later locale frame-of-reference after you left, you wouldn't be saying "this is what will happen four years from now" you would be saying "this is what is happening at alpha centauri concurrent with the last 10 minutes that we will observe four years from now."

Grey_Wolf_c
2018-11-14, 02:20 PM
But why does FTL have to use the originator's frame of reference? Why wouldn't it use it's own FoR that is based on the FTL medium / whatever you travel through to make the trip?

Because no matter what, every Frame of Reference can tell their version of what just happened, and if FTL is involved, some FoR will tell you that things happened in a different order from what the other FoR saw.

In the thread link earlier, Douglas has a simple example (http://www.giantitp.com/forums/showsinglepost.php?p=22762284&postcount=44) of how FTL messaging will break causality. He does a much better job of explaining it that I ever can, so I thoroughly encourage you to read it.

ETA: What the keck, I'll give it a try.

There is a fueling station set 1 light Year away from Earth (the fueling station constantly matches speeds with Earth, somehow). A spaceship travelling at 50% the speed of light passes past it without stopping. But a guy at the station manages to take a picture of the space ship as it passes. The picture shows the spaceship rushing past, all its external lights blazing - which is bad, because that's a waste of energy.

That guy then sends the picture to Earth by means of his trusty FTL Messenger System. It arrives instantly, and someone on Earth then sends it to the spaceship with instructions to turn off the lights. When the captain sees the picture, he smacks his forehead and orders the lights turned off.

Now, let's examine when this happens.

From the FoR of the station, Earth is 1 LY away. So, when the message is sent, it arrives 1 year before the light from the the station arrives to Earth. So it gets sent back to the spaceship, who receives it 1 year before its light is due to arrive to Earth.

But the ship is travelling at 50% the speed of light. By the time it passes the station, it is less that a light year away from Earth, because from it's FoR, Earth is hurtling towards it at 50% the speed of light, getting closer and closer every second (so it's, say, about 6 light months away, which it'll take about 9 months to cover*). So where was the ship one light year before it arrived? Why, it hadn't reached the fueling station yet, nor would it for, say, about 4 more months. So, they turn off the lights, and four months later they pass the fueling station, where the photo is taken... except the photo doesn't show the lights on, so it doesn't get sent to Earth, and no-one tells the captain, so the lights stay on... and there goes causality.

Grey Wolf

*This is called length contraction, and it is, as you can now see, a hell of a drug.

Frozen_Feet
2018-11-14, 04:07 PM
The entirety of the problem is precisely that there is no constant, universal frame of reference. If writing fiction, sure, you can arbitrarily state one exists, but as far as reality and known physics go... there isn't any, and there is no plausible form of FTL that would come with such a thing.

Because of this lack of universal frame, FTL creates events where information travels backwards between two (or more) frames, that is, causes precede effects.

Now, there are interpretations of relativity, string theory etc. where these acausal interactions aren't a big deal. But those interpretations also imply some seriously weird stuff which remains unfalsifiable at the moment.

What kind of weird stuff? Oh, just eternalism and time being an illusion, near-infinite branching universes which don't meaningfully interact, non-existence of free will, etc.

Kato
2018-11-14, 04:40 PM
I don't think I can give a more concis answer than has already been given but I'll try for a very short and simpler (???) one. (It's really quite hard to give a simple, complete answer on this I think)

The problem is relativity and its lack of a universal time frame. Teleportation is cheating and doesn't matter here, it's moving while obeying the laws of relativity. The problem is, stuff is always moving so "just ignore moving objects" is not a helpful answer. If you move or just send a message FTL, a moving observer due to their different perception of time might see you or your message arrive before you send it. Which instantly leads to the possibility of them telling you, you need to get going / send the message (or not). Dang, causality broken.


Because it really is something that has nothing to do with what we see in our daily lives I can absolutely understand this is hard to grasp. Honestly, without decent, convincing diagrams (or formulas if you're that kind of person) I would not expect most people, me included, to entirely understand it. Here's an easy-ish explanation... Thanks, google. (http://www.physicsmatt.com/blog/2016/8/25/why-ftl-implies-time-travel)


(edit: and before someone tries to argue with me, I debated against this in the other thread, I was always trying to make it clear I know it requires ditching relativity or cheating in some other way and I never claimed FTL was realistic, just not that hard to imagine :smalltongue:)

Gallowglass
2018-11-14, 05:06 PM
Because no matter what, every Frame of Reference can tell their version of what just happened, and if FTL is involved, some FoR will tell you that things happened in a different order from what the other FoR saw...

*This is called length contraction, and it is, as you can now see, a hell of a drug.

I mean, you are probably right and I am probably wrong because I have certainly heard this argument before. But it never made sense to me.

So you have a station stationary at 1 LY from earth. Somehow they use some magic to keep it exactly and forever the space distance needed for light to get from the station to earth in 1 year at 186,282 miles per second.

A vessel passes the station moving at .5 C (186,282/2 miles per second) meaning at the moment it passes the station it will take 6 months for it to get to earth.

Some guy happens to take a picture at the EXACT moment both the station and the ship are at the 1 LY to earth mark. We are assuming a magic camera that takes a picture without light so he gets the picture you are positing.

Seconds to minutes are passing for both the station, the ship and the earth. The person takes the picture, sees the problem, walks over to the FTL message box and sends the message. For all three PoR we'll say a minute passes.

The station is still 1 LY from earth. Earth is still 1 LY from the station. The ship is now 1LY-1Lmi from Earth. Earth is now 1LY-1Lmi from the ship. So the SHIP is now 186,282*60 miles closer to earth than the station is.

The guy on earth gets the message, sees it, decides what to do and punches in the ship code and forwards the message. That takes another minute. At the moment he sends the message the station is still 1 LY from earth, Earth is still 1 LY from the station, but the ship is now 186,282*120 miles closer to Earth than the station and is 1LY-2Lmi from Earth and Earth is 1LY-2Lmi from the Ship.

When the ship gets the message, its moved 186,282*120 miles from the point its picture was taken. Its now that much closer to earth than when the picture was taken. Its 1LY-2LMi from earth.

I don't get why you think that the message would get to the ship WHILE it was still 1 LY from Earth just because that was when the picture was taken. Time has progressed for all three FoR.

This is assuming the instantaneous transport of the message. IF the message moved at some multiple of C then it gets messier but it still just adding time to each frame of reference based on how long each discreet activity takes.

Of course, as I said, this is most likely a failure of my imagination.

Grey_Wolf_c
2018-11-14, 06:18 PM
The station is still 1 LY from earth. Earth is still 1 LY from the station. The ship is now 1LY-1Lmi from Earth. Earth is now 1LY-1Lmi from the ship. So the SHIP is now 186,282*60 miles closer to earth than the station is.

This is only true from the PoV of the station. From the PoV of the ship, Earth is significantly closer. I don't know the exact math, but it's something like 8-9 light months. What you are suggesting - that distance is equal for all frames of reference - violates General Relativity, and has been proven to not be the case. The only thing that is identical for all frames of reference is the speed of light. Not time, and not distance.

ETA: by the way, the example could work for a station that was moving with respect to Earth. I had it relatively immobile just to simplify the situation.

Grey Wolf

keybounce
2018-11-14, 06:26 PM
The guy on earth gets the message, sees it, decides what to do and punches in the ship code and forwards the message. That takes another minute. At the moment he sends the message the station is still 1 LY from earth, Earth is still 1 LY from the station, but the ship is now 186,282*120 miles closer to Earth than the station and is 1LY-2Lmi from Earth and Earth is 1LY-2Lmi from the Ship.

When the ship gets the message, its moved 186,282*120 miles from the point its picture was taken. Its now that much closer to earth than when the picture was taken. Its 1LY-2LMi from earth.

I don't get why you think that the message would get to the ship WHILE it was still 1 LY from Earth just because that was when the picture was taken. Time has progressed for all three FoR.

This is my thinking as well.

I thought I mention in my first post that if you can make multiple jumps in different frames, you will get time travel/causality violation because the different light cones are at different angles (or something like that).

But if there is a special reference frame?

The universe itself is a non-moving, non-rotating reference frame -- perhaps that is special?

(It's one thing to say that all observers see the same laws of physics -- that's confirmed.
It is something else to say that you cannot detect a privileged frame -- that's an assumption, at best we can say that we don't know how to detect one.
It is something else entirely to say that a privileged frame cannot exist -- there's nothing to support that.)

Douglas
2018-11-14, 06:27 PM
I mean, you are probably right and I am probably wrong because I have certainly heard this argument before. But it never made sense to me.

So you have a station stationary at 1 LY from earth. Somehow they use some magic to keep it exactly and forever the space distance needed for light to get from the station to earth in 1 year at 186,282 miles per second.

A vessel passes the station moving at .5 C (186,282/2 miles per second) meaning at the moment it passes the station it will take 6 months for it to get to earth.

Some guy happens to take a picture at the EXACT moment both the station and the ship are at the 1 LY to earth mark. We are assuming a magic camera that takes a picture without light so he gets the picture you are positing.

Seconds to minutes are passing for both the station, the ship and the earth. The person takes the picture, sees the problem, walks over to the FTL message box and sends the message. For all three PoR we'll say a minute passes.

The station is still 1 LY from earth. Earth is still 1 LY from the station. The ship is now 1LY-1Lmi from Earth. Earth is now 1LY-1Lmi from the ship. So the SHIP is now 186,282*60 miles closer to earth than the station is.

The guy on earth gets the message, sees it, decides what to do and punches in the ship code and forwards the message. That takes another minute. At the moment he sends the message the station is still 1 LY from earth, Earth is still 1 LY from the station, but the ship is now 186,282*120 miles closer to Earth than the station and is 1LY-2Lmi from Earth and Earth is 1LY-2Lmi from the Ship.

When the ship gets the message, its moved 186,282*120 miles from the point its picture was taken. Its now that much closer to earth than when the picture was taken. Its 1LY-2LMi from earth.

I don't get why you think that the message would get to the ship WHILE it was still 1 LY from Earth just because that was when the picture was taken. Time has progressed for all three FoR.

This is assuming the instantaneous transport of the message. IF the message moved at some multiple of C then it gets messier but it still just adding time to each frame of reference based on how long each discreet activity takes.

Of course, as I said, this is most likely a failure of my imagination.
There's an implicit assumption in here that there is only one correct value for "what time is it on Earth right now" at the location of the station and ship. That assumption is incorrect - the answer for "what time is it on Earth right now" depends on how quickly your frame of reference is moving towards Earth (or rather, how quickly Earth is moving towards the observer in your frame of reference). The station and the ship will give two different answers to this question - and they're both right.

That fundamental disagreement over what the current time is at a distant location is the core of the causality violation. Take that, a means of FTL communication, and a way to switch which reference frame the FTL comm is using (just accelerating the sender and/or receiver should do it), and those three ingredients add up to a way to make a message arrive back at its origin before it was sent.

Frozen_Feet
2018-11-14, 07:09 PM
For all three PoR we'll say a minute passes.

This right here. This is the false assumption you're making. Precisely because there is no constant universal reference frame, a minute on the ship is not equal to minute on the station is not equal to minute on Earth. True simultaneity does not exist. Once you introduce FTL, the math actually works out to negative time passing in some frames in relation to others. To give a rough idea, your "instant messaging" device would need tachyons or similar hypothetical particles to work.

jayem
2018-11-14, 07:28 PM
I mean, you are probably right and I am probably wrong because I have certainly heard this argument before. But it never made sense to me.


There is an extent where you are begging the question with the phrasing. In particular if we assume the Lorentz contractions then we need to be careful when we say distances and times as you do in the paragraph. So you keep 'causality' at the expense of '(einsteinian) relativity', and because you don't look at anything to do with c you don't see it.

For the moment lets instead assume Gallileon relativity (I.E distances/times are the same for all observers). On receipt of the signal earth instantly radio's back to the ship.

From the frame of the station, the ship has moved 1/3 of the distance. the radio beam has moved 2/3 of the distance. The radio beam has moved 2/3Light Years in 2/3 Years.

From the frame of the ship, the ship is stationary. The moving earth sends the radio wave when it is 1 LY away, it then 'chases' after the wave at 0.5C. After 2/3 Years the light wave hits the ship, having traveled 1 LY.

Now here we have two options.

One is to say of course the light moves faster, it was fired from a 'moving planet'. When we move to the ships frame we need to take that into account the light is moving at 1.5C in ship frame. And all is hunky doory as relates to the different frames (which is nice for assorted reasons), everything makes perfect sense.
However when we look at light in terms of electric and magnetic fields you end up with a hideous mess, which I can't even begin to fathom.

The other is to say that the earth-station frame is special. We expect things to break when looking at ship view. At first sight again this looks fine, things make sense in the only frame that matters.
However conservation and symmetry laws are closely related, in particular conservation of momentum is gone and Newtons 2nd law is some ghastly mess.

At this point Einstein took a third option. Basically to do what Galileo did for speed (I.E our first option) to distance and time as well, so that electo-magnetism can work. Unfortunately it's not as simple as adding it example.

The problem is we now now need to rework your example (we'll do a half assed version where we fix distances but have a constant time, this breaks when we add a third frame, einstein & lorentz did what they did for a reason)

So from the station/earth frame (radio version)
They are 1LY apart, and the spaceship meets the radio (traveling at C) at 2/3s year time (all is ok the ship Has moved 1/3 LY, the beam 2/3 LY all add up.)
In the ship frame the 'stationary' spaceship meets the radio (traveling at C) at 2/3s year time, it must have been 2/3 of a light year away.
(in proper relativity the difference is split between time and space, here we have exaggerated length contraction without time dilation).
Now switching back to your example with those distances



So you have a station stationary at 1 LY from earth. Somehow they use some magic to keep it exactly and forever the space distance needed for light to get from the station to earth in 1 year at 186,282 miles per second.

A vessel passes the station moving at .5 C (186,282/2 miles per second) meaning at the moment it passes the station it will take 6 months for it to get to earth.
[as before]




So you have a station stationary at 0.666 LY from earth. Somehow they use some magic to keep it exactly and forever [this bit doesn't work, because we didn't do the job properly].

A station passes the vessel moving at .5 C (186,282/2 miles per second). At the moment it passes the station it will take 6 months for earth also to pass it.

[in this case because we've ignored the time aspect, causality is also maintained]






So you have a station stationary at 0.866 LY from earth. Somehow they use some magic to keep it exactly and forever so light takes 0.866 Y to reach it.

A station passes the vessel moving at .5 C (186,282/2 miles per second). At the moment it passes the station it will take 6 months for earth also to pass it.

Actions that took a minute on the ship in earths frame now take 0.866 minutes.
Actions that took a minute on the earth in earths frame now take 1.15 minutes

And it is with that difference that you can play around with causality. In this example if you had two alarms that start and stop at the same time from earths frame, now the ship one now stops nearly half a minute earlier. If you tie that with further actions then things get messy

Anymage
2018-11-15, 06:34 PM
This is my thinking as well.

I thought I mention in my first post that if you can make multiple jumps in different frames, you will get time travel/causality violation because the different light cones are at different angles (or something like that).

But if there is a special reference frame?

In fiction, this is a wonderful idea. Relativity is weird and unintuitive, and handwaving it for space opera simplifies a lot.

In reality, this has been experimentally disproven. Repeatedly.


The universe itself is a non-moving, non-rotating reference frame -- perhaps that is special?

(It's one thing to say that all observers see the same laws of physics -- that's confirmed.
It is something else to say that you cannot detect a privileged frame -- that's an assumption, at best we can say that we don't know how to detect one.
It is something else entirely to say that a privileged frame cannot exist -- there's nothing to support that.)

"The universe" doesn't have a clear center or preferred direction. (Everybody actually does see themselves at the exact center of the universe and thinks that everything is moving away from them at an ever increasing speed, but that's more relativistic weirdness rather than less.) Just looking at nearby galaxies, is there any reason why the Milky Way would have the right of it as opposed to the Andromeda galaxy that's coming towards us at ~68 miles per second?

And while you're right that I can't prove for certain that it isn't just mischievous pixies screwing with particles, or that you're just a brain in a jar and everything from experimental data to other people you interact with are just data fed in by your handlers, relativity makes many predictions that have been consistently verified. If you want to overturn it with "I dunno man, how can we really know for sure", create a new model that makes its own predictions and then get to work testing them.

Lethologica
2018-11-15, 07:06 PM
This even suggests a way to implement that wonderful idea for fiction. Simply have FTL that is experimentally verified to only work in a particular reference frame for whatever science-fictional reason we feel like, leave everything else unchanged, and hey presto. We'll also need some way to get to relativistic speeds, so as to interact with interesting local objects as something other than a smear on the windshield, but that's no big deal.

Darth Credence
2018-11-16, 10:48 AM
Lethologica - that sounds similar to the method used in Robert Kroese's Rex Nihilo books. IIRC, the idea is that there are infinite reference frames, and you create one where you are actually close to where you want to go, then reverse engineer an entire frame around that fact. Then you input that as the new reference frame for the journey, and you can travel interstellar distances quickly. It is a comedy book, so the idea isn't exactly fleshed out, and it may have been referring to changing geometry rather than dealing with relativity at all.

Grey_Wolf_c
2018-11-16, 03:10 PM
The universe itself is a non-moving, non-rotating reference frame -- perhaps that is special?

Then the laws of physics as we understand them don't apply, and your original question is moot. Sure, you can imagine a universe in which Newtonian Physics apply. But in that universe, the speed of light isn't special. Going FTL just requires a sufficient amount of thrust. The speed of light would be like the speed of sound.

It'd also, I'm sure, cause all kinds of weird consequences, which is why Einstein figured out that something had to give, and it wasn't the speed of light, but you could simply hand wave that away.


(It's one thing to say that all observers see the same laws of physics -- that's confirmed.
It is something else to say that you cannot detect a privileged frame -- that's an assumption, at best we can say that we don't know how to detect one.
It is something else entirely to say that a privileged frame cannot exist -- there's nothing to support that.)

No, actually it is the other way around: physicist say there can't be a privileged frame of reference because otherwise not all observers would see the same laws of physics. For example, it could be that observers moving towards the light would measure the speed of light as being slower than observers at rest. Therefore we could find that absolute rest state by adjusting our speed until it maximized the speed of all incoming light.

Grey Wolf

Tvtyrant
2018-11-16, 03:35 PM
Relativity as I understand it is based on the imbalance between the universality axiom, ie the unprovable assumption that the laws of physics remain the same and are the same everywhere.

We actually know that axiom is false, because of the Big Bang. The problem is that you can't measure changes in the laws of physics because observations become unrepeatable. Science is entirely dependent on that axiom, so if something or somewhen actually did work differently we wouldn't know.

Grey_Wolf_c
2018-11-16, 04:11 PM
Relativity as I understand it is based on the imbalance between the universality axiom, ie the unprovable assumption that the laws of physics remain the same and are the same everywhere.

[citation needed]

I suppose in broad strokes all science is based on that axiom, but not relativity in particular. Relativity is based on the proven observation that the speed of light is identical for all observers regardless of relative speeds.

Grey Wolf

Mightymosy
2018-11-17, 10:03 AM
I feel a lot of people miss a core principle of physics:

What IS physics?

Physics is a system of mathematical equations which try to make sense of the universe we observe around us - put it into a theoretical framework so me might try to understand it better.
Also, try to use these equations to predict how certain things in the future will turn out. And use these equations to construct machines and other devices to use the "laws" of the universe for our advantages.

Now, relativitiy theory is an awesome theory. An awesome theory that was discarded at first, until a lot of very intelligent people were ultimately proven wrong in their pre-relativistic worldview.

But kept in perspective, relativistic theory was just ONE step in the long history of physics, ultimately just one other big theory that expanded upon the ones before it.

And if you ask me: It won't be the last, either.


In other words: physics theory is best at describing things we know. Put it into equations. I would not trust physcis to predict EVERYTHING that is impossible. In fact, the historical development went in cycles: each cycle someone discovered something that went against the current equations, against the current theory. And then devised a new all-encompassing theory that explained everything better. But so far, none of them had proven to last forever. Some day, I expect someone will discover a phenomenom that breaks the concept of relativistic theory. And then people will come up with another theory that us better suited to describe reality as we - or they - know it.


The only constant in the history of humankind is that every single one who thought they knew everything has been proven wrong ultimately.

Lvl 2 Expert
2018-11-17, 11:43 AM
But kept in perspective, relativistic theory was just ONE step in the long history of physics, ultimately just one other big theory that expanded upon the ones before it.

And if you ask me: It won't be the last, either.

But that doesn't mean it's wrong in this aspect, or any more wrong than any alternative. Quite the opposite, it's the best model for the way the universe at this scale works that we currently have.

I'd highly recommend reading the thread linked earlier (http://www.giantitp.com/forums/showthread.php?548530-FTL-and-Violating-Causality&highlight=FTL+causality). It's complicated, I already forgot most of what I learned from it and how this all worked, but theresult of it is this: Any form of information transfer* faster than the speed of light can result in you receiving your own message before you sent it. I think back then I figured out a way to do it with a fast spinning space station with an instant messaging antenna on one end. (It still works even if the messaging is not instant, that just makes this a bit easier to think about.) And if you can send a message even a millisecond back in time it stands to reason you can just repeat the process to receive any information you will ever send right on the moment your machine was started up.


* Quantum entanglement has an effect that goes faster than the speed of light, but the only way to know the effect has taken place and get any kind of information out of it is to have a second slower than light information channel running alongside it. That's kind of creepy in its own right, it seems almost designed to fit the rules of the universe, rather than having a good physical reason for doing what it does.

Roland St. Jude
2018-11-17, 12:01 PM
I was thinking the same thing, but figure it's barely acceptable because of the media impact of it. But I'm flexible on it.Sheriff: That's a fair judgment.

I should have read through the thread before I moved it. :smallamused: Oh well, what's done is done.

Anymage
2018-11-17, 01:03 PM
In other words: physics theory is best at describing things we know. Put it into equations. I would not trust physcis to predict EVERYTHING that is impossible. In fact, the historical development went in cycles: each cycle someone discovered something that went against the current equations, against the current theory. And then devised a new all-encompassing theory that explained everything better. But so far, none of them had proven to last forever. Some day, I expect someone will discover a phenomenom that breaks the concept of relativistic theory. And then people will come up with another theory that us better suited to describe reality as we - or they - know it.

The only constant in the history of humankind is that every single one who thought they knew everything has been proven wrong ultimately.

Einstein didn't just say that Newton was a fool and that everything Newtonian was wrong. He had to work within the observations that Newton worked quite well within familiar parameters, and only started breaking down as you got far outside of those.

We know that our current model of the universe has some very deep flaws. We even have a few geeks hypothesizing ways to go faster than light while within our current understanding of physics. How these pan out will inform future models of physics. But we can only answer questions to the best of our current understanding now. And "here's what our current understanding says" is a lot more productive than "well, you can never know for absolutely sure..."

factotum
2018-11-17, 02:02 PM
Yeah, I agree. Newtonian mechanics are not "wrong" per se--they describe the universe pretty well so long as you're not going very fast. Einstein's work added to Newton, it didn't entirely supersede it. Anything that comes along which describes the universe better than Einstein similarly won't likely replace relativity but add more accuracy to the corner cases.

keybounce
2018-11-17, 09:56 PM
I said: "The universe itself is a non-moving, non-rotating reference frame -- perhaps that is special?"


Then the laws of physics as we understand them don't apply, and your original question is moot. Sure, you can imagine a universe in which Newtonian Physics apply. But in that universe, the speed of light isn't special. Going FTL just requires a sufficient amount of thrust. The speed of light would be like the speed of sound.


One of us is not understanding the other here.

General relativity takes as an assumption that the laws of physics appear the same to all observers. (Before you get all techie on me, yes I know that's only approximately what it says). From that assumption, it derives some numerical predictions, and those predictions match actual observation.

All this means is that the fundamental assumption -- that the laws of physics appear the same to everyone -- is probably close to accurate.

It is possible to add the axiom, "there exists a privileged reference frame" to relativity without disturbing either of the existing 2 axioms. You will still get the same behaviors -- special relativity's apparent change in observed length and passage of time, general relativity's space-time, etc.

I said:


(It's one thing to say that all observers see the same laws of physics -- that's confirmed.
It is something else to say that you cannot detect a privileged frame -- that's an assumption, at best we can say that we don't know how to detect one.
It is something else entirely to say that a privileged frame cannot exist -- there's nothing to support that.)



No, actually it is the other way around: physicist say there can't be a privileged frame of reference because otherwise not all observers would see the same laws of physics. For example, it could be that observers moving towards the light would measure the speed of light as being slower than observers at rest. Therefore we could find that absolute rest state by adjusting our speed until it maximized the speed of all incoming light.

Grey Wolf

Please explain how the existence of a privileged frame of reference implies that the assumptions of general relativity cannot hold.

Jasdoif
2018-11-18, 01:47 AM
I said: "The universe itself is a non-moving, non-rotating reference frame -- perhaps that is special?"
....
General relativity takes as an assumption that the laws of physics appear the same to all observers.General relativity is based on (and named after) the general principle of relativity; that the laws of physics are the same in all frames of reference.

A special frame of reference such as you describe would necessarily vary on those laws of physics which make it special; therefore physical laws would not be the same in all frames of reference, therefore general relativity would not hold.

Brother Oni
2018-11-18, 07:38 AM
For all three PoR we'll say a minute passes.


This right here. This is the false assumption you're making. Precisely because there is no constant universal reference frame, a minute on the ship is not equal to minute on the station is not equal to minute on Earth. True simultaneity does not exist. Once you introduce FTL, the math actually works out to negative time passing in some frames in relation to others. To give a rough idea, your "instant messaging" device would need tachyons or similar hypothetical particles to work.

As a biochemist, I too have trouble wrapping my head around the mechanics of how time passes differently at relativistic speeds, until I saw this Christmas lecture by Brian Cox: "Is time travel possible? The Science of Dr Who" (https://www.youtube.com/watch?v=-O8lBIcHre0).

Since the speed of light is fixed, the passage of time is personal to an individual (ie to a reference frame) as Frozen_Feet said.

Time passing differently the faster you go isn't just a scientific curiosity, it's a fundamental problem that has ramifications for today's technology - GPS satellites have a timing adjustment specifically for relativistic effects: wiki link (https://en.wikipedia.org/wiki/Error_analysis_for_the_Global_Positioning_System#R elativity).

shawnhcorey
2018-11-19, 01:57 PM
To understand why FTL flight violates causality and is time travel can be explain easily with light cones (https://en.wikipedia.org/wiki/Light_cone). I'm not going to explain what light cones are but here are some consequences of them.

If there are two events, A and B, such that B is in A's future light cone, then A is in B's past light cone. An observer, by changing their velocity, can change the distance and time between A and B but their order never changes. B is always in A's future light cone and A is always in B's past light cone. Note that A need not cause B. They may be independent of each other.

If there are two other events C and D such that they are not in each other's light cones, then an observer, by changing their speed, can change the order of the events. For half of a possible velocities, C will come before D and for half, D will come before C.

Now to look at FTL flight. You enter FTL flight (event C) and leave it some distance away (event D). These two events are outside of each other's light cone because of FTL flight. The FLT flight causes them to be outside of each other's light cones. In your initial frame of reference (before the flight) C happens before D.

Now that you have arrive at your destination (after D), you change your velocity so that C is in the far future. This violates causality. Your arrival happens before its cause, that is, your departure.

Now you use FLT flight to jump back to your starting location. But C, your departure is in the far future. You have travelled back in time.

So, FTL flight both violates causality and can be used for time travel.

Mightymosy
2018-11-23, 09:08 PM
Einstein didn't just say that Newton was a fool and that everything Newtonian was wrong. He had to work within the observations that Newton worked quite well within familiar parameters, and only started breaking down as you got far outside of those.

We know that our current model of the universe has some very deep flaws. We even have a few geeks hypothesizing ways to go faster than light while within our current understanding of physics. How these pan out will inform future models of physics. But we can only answer questions to the best of our current understanding now. And "here's what our current understanding says" is a lot more productive than "well, you can never know for absolutely sure..."

Well, Newton described how matter moved in our universe, and Einstein showed that Newton's rules weren't quite right in all circumstances.

But your last sentence is actually exactly what I mean:

The sentence "FTL speed is impossible and incompatible with physics" is a much worse sentence than "TFL speed seems impossible to do with the methods we have now and with the current understanding of interstellar physics we have now, but you can never know for sure".

NichG
2018-11-23, 09:51 PM
Well, Newton described how matter moved in our universe, and Einstein showed that Newton's rules weren't quite right in all circumstances.

But your last sentence is actually exactly what I mean:

The sentence "FTL speed is impossible and incompatible with physics" is a much worse sentence than "TFL speed seems impossible to do with the methods we have now and with the current understanding of interstellar physics we have now, but you can never know for sure".

I'm more bothered by the confident assertion that it is impossible to know, than the confident assertion that a particular specific thing has been shown to be nonsensical. The degree of surety matters, and your second sentence hides that by essentially equating a 10^-80 uncertainty to a 1% uncertainty.

Based on the evidence, we should probably assign more confidence to the statement 'faster than light transmission of information would lead to non-directed causal structures' than the statement 'I will be alive tomorrow'. For example, in the case of a particle physics experiment collecting ~10^10 total events (about 15 years worth of runtime), this would correspond to any observation better than ~6 sigma.

Mightymosy
2018-11-24, 03:24 AM
It is hard to assign actual numbers to probabilities of things you consider "impossible".

Either we think they are impossible with our current understanding, or we don't.

Expressing that as numbers doesn't make much sense to me.

"FTL seems impossible with the methods we have, and the understanding of interstellar physics we have" is good enough, if you ask me.

It says that we can not do it right now, but we can not know if it is impossible in the future, but to be possible it would require new methods and a partially new understanding of space physics.

I wouldn't put any numbers into it.

NichG
2018-11-24, 03:41 AM
It is hard to assign actual numbers to probabilities of things you consider "impossible".

Either we think they are impossible with our current understanding, or we don't.

Expressing that as numbers doesn't make much sense to me.

"FTL seems impossible with the methods we have, and the understanding of interstellar physics we have" is good enough, if you ask me.

It says that we can not do it right now, but we can not know if it is impossible in the future, but to be possible it would require new methods and a partially new understanding of space physics.

I wouldn't put any numbers into it.

A scientific theory doesn't just rest on whether people feel that it makes sense or is compelling. Generally, it has to provide some predictive or explanatory power which is then backed up by experimental tests. Those tests have some distribution of results, and errors or fluctuations can occur - by characterizing those, we can construct an error model, and thereby ask the question 'if we were to repeat all of these experiments, what is the probability that we would come to a different conclusion?'

That number, for physics experiments, tends to be extremely small. Something like 10^-80 is actually not out of the question for measurements that are easy to repeat or produce large amounts of data.

It's not about thinking things are possible or impossible, it's about establishing the constraints which any theory must satisfy in order to be consistent with what we have observed. So relativity had to fit inside the space carved out by Newtownian mechanics experiments, and whatever might replace relativity has to fit inside the space carved out by relativity experiments. While it's possible for the experiments to have been mistaken, it's not the case that we have no way of assigning numbers to the probability that they were mistaken.

shawnhcorey
2018-11-24, 07:23 AM
FTL is impossible because every observer measures the speed of light in a vacuum to be the same. FTL would not just break our understanding of the universe; it would break the universe. Most people do not realize how important the fundamental constants are and how closely coupled to each other they are. FTL would mean nothing is possible any more.

Anymage
2018-11-24, 10:04 AM
Well, Newton described how matter moved in our universe, and Einstein showed that Newton's rules weren't quite right in all circumstances.

But your last sentence is actually exactly what I mean:

The sentence "FTL speed is impossible and incompatible with physics" is a much worse sentence than "TFL speed seems impossible to do with the methods we have now and with the current understanding of interstellar physics we have now, but you can never know for sure".

I have an idea that next time you're sick, you let me open fire at you with a machine gun under the expectation that only the unhealthy cells will be hit. Sure, every time the "people getting shot" experiment has been conducted the result has been severe injury or death, but this time it might work out perfectly and in fact grant superpowers.

Whattya say? After all, medical science is changing all the time, so we can never know exactly for sure.

Grey_Wolf_c
2018-11-24, 06:38 PM
Well, Newton described how matter moved in our universe, and Einstein showed that Newton's rules weren't quite right in all circumstances.

Newton knew that his laws and equations weren't right in all circumstances. Mercury was a dead giveaway.

Science progresses by reducing the number of scenarios it cannot explain & predict. At this point, the speed of causality of the Universe is pretty well established and tested. Pretending that because science doesn't know everything, we can assume that what it does know can be doubted is NOT how science works.

Grey Wolf

Jay R
2018-11-24, 07:19 PM
I said: "The universe itself is a non-moving, non-rotating reference frame -- perhaps that is special?"

No, it isn't.

I can define a reference frame in which the solar system is not moving or rotating.
I can define a reference frame in which the Milky Way is not moving or rotating.
I can define a reference frame in which the Local Group containing the Milky Way is not moving or rotating.
I can define one in which the Laniakea Supercluster (which includes the Local Group) is not moving or rotating.
I can define one in which the Centaurus Great Wall (which includes the Laniakea Supercluster) is not moving or rotating.
I can even define one in which the center of all currently known extra-galactic structures is not moving or rotating.

But I cannot define "The universe itself" and have any idea what that frame of reference is.

There is no single reference frame that is "The universe itself".


Please explain how the existence of a privileged frame of reference implies that the assumptions of general relativity cannot hold.

Turn it around. The assumptions of general relativity say that if you define a frame of reference, for instance "The universe itself", then I can define another one that moves relative to that one, and which is just as definitive as that one.

Your frame of reference isn't privileged. It just isn't. It's as arbitrary as any other. If an object is moving at 0.5c in your reference frame, then whatever appears motionless to you is traveling at 0.5c to that reference frame.

The same equations needed to refer to another reference frame from yours will work to refer to yours from that one.

Lvl 2 Expert
2018-11-25, 04:33 AM
This is an idea people have had before. They figured that if there is a privilidged reference frame it probably isn't locked to the surface of the earth. So they (I think) went about measuring the speed of light in several directions in several parts of our yearly cycle. Relativity is the result of these experiments failing to detect any changes. If there is no universal right frame of reference, but we still have a universal top speed, then time must be interacting with our reference frame somehow.

gomipile
2018-11-25, 03:06 PM
Here's a relatively(heh) good video on this topic:


https://youtu.be/HUMGc8hEkpc

Kato
2018-11-26, 03:58 AM
This is an idea people have had before. They figured that if there is a privilidged reference frame it probably isn't locked to the surface of the earth. So they (I think) went about measuring the speed of light in several directions in several parts of our yearly cycle. Relativity is the result of these experiments failing to detect any changes. If there is no universal right frame of reference, but we still have a universal top speed, then time must be interacting with our reference frame somehow.

Minor note(s) :
You're probably referring to the famous (among physicists) Michelson - Morley - experiment which indeed proved that lightspeed is independent of the movement of Earth, although it was meant to prove the existence of the Aether (which would be kind of that special reference frame)
Also, this is a great instance of an experiment proving scientists wrong and them not going 'oops, better do something else' but 'dang, time to change our opinions on this'.

Lvl 2 Expert
2018-11-26, 04:36 AM
Minor note(s) :
You're probably referring to the famous (among physicists) Michelson - Morley - experiment

I didn't know the name, I couldn't find it by googling for aether and I left the comment rather quickly on my phone. Thanks for the added information!

Rakaydos
2018-11-26, 01:11 PM
Question for the high level physics people.

So, all the lightcone diagrams I've seen are flat. But any number of FTL methods in fiction "Pinch", "Warp" or "wormhole" from one point in spacetime to another.

Assuming such an FTL, able to travel outside it's lightcone, but assume power requirements go asymptotic as it approaches a causality violation with anything that was in it's pre-jump lightcone.

So you detect a ship jumping in, telling you there's a disaster where they jumped from, two years from now. You can jump back there, with relief supplies at any time up to the moment the messanger ship left, two years AFTER it arrived, (though the power requirement goes up the closer you cut it) but no sooner, even though in a hypothetical universal frame, it hasnt happened yet. If another ship arrives or leaves, it resets the earliest arrival time, and if you wait long enough for the light of the disaster to arrive, you cannot violate what you could theoretically see happen.

Does this thoought expiriment hold up?

DavidSh
2018-11-26, 02:06 PM
...

Does this thoought expiriment hold up?
A causality violation in this sort of model looks like a loop of hops, where each hop terminates in the past light cone of the next hop. In a two-hop loop, the first hop might go from time 0 at location 0 to time 10 at location 1000, and the second hop might go from time 20 at location 1000 to time -10 at location 0. (Coordinates as given by one observer.)

It looks like you are proposing making one of the hops in such a loop to be physically forbidden. This sounds superficially plausible, but there are some issues.
1) Which hop should be forbidden? You can construct cases that appear, from a carefully chosen frame of reference, to be perfectly symmetrical. E.g., maybe from one reference frame the first hop goes from location -100 at time 0 to location 100 at time -1, and the second hop from location 100 at time 0 to location -100 at time -1.
2) You can also use such a process to send 1 bit of information into the past, if you are willing to set things up in advance. Tell your distant partner to send you a message only if it doesn't get any message from your future. For example, you are going to do an experiment that might kill you. Arrange your distant partner to send a message telling you to stop, unless they get a message from your later self saying that you survived unscathed.

Rakaydos
2018-11-26, 05:02 PM
A causality violation in this sort of model looks like a loop of hops, where each hop terminates in the past light cone of the next hop. In a two-hop loop, the first hop might go from time 0 at location 0 to time 10 at location 1000, and the second hop might go from time 20 at location 1000 to time -10 at location 0. (Coordinates as given by one observer.)

It looks like you are proposing making one of the hops in such a loop to be physically forbidden. This sounds superficially plausible, but there are some issues.
1) Which hop should be forbidden? You can construct cases that appear, from a carefully chosen frame of reference, to be perfectly symmetrical. E.g., maybe from one reference frame the first hop goes from location -100 at time 0 to location 100 at time -1, and the second hop from location 100 at time 0 to location -100 at time -1.
2) You can also use such a process to send 1 bit of information into the past, if you are willing to set things up in advance. Tell your distant partner to send you a message only if it doesn't get any message from your future. For example, you are going to do an experiment that might kill you. Arrange your distant partner to send a message telling you to stop, unless they get a message from your later self saying that you survived unscathed.

1) So, to expand- The FTL would cost more energy to use the closer it gets to the light cone. So the first jump in the universe, the most energy efficent jump is a T=0 translation- equally distant from the future and past time cones. But any reply (being by definition some miniscule amount of time, locally, after recieving the message) would have a minimum energy to target a time halfway between when the message was sent, and when the light from the message would be seen- effectively, traveling at 2c. You can spend more energy to get the reply closer to the time the message was sent from, but to send it back to the same instant it left costs infinite energy. It also dosent matter how long you wait to send the reply (as long as you dont wait so long the light from the original message catches up with you), it only matters to your power requirements where the light cone is in relation to your destination.

This also applies to any FTL arrival "seen" by the message before it was sent. If station 1 sent a message to station 2s past, station 2 could send it to staion 3's past, but station 3 could only send it back to station 1's future.

2) I may be channeling Whaetly from Portal here, (This statement is false? Hmm, True! That was easy) but it would seem to me the natural course is that the partner never recieves a message telling them to not send a message, because the message they will send blocks you from telling them not to send it. Whether you succed at your expiriment or not, if you cant transmit, you will have recieved the stand down command, and the stand down command keeps you from transmitting.

Anymage
2018-11-26, 06:02 PM
You might as well institute a paradox system directly into your setting, a'la Continuum (https://en.wikipedia.org/wiki/Continuum_(role-playing_game)). Fictionally speaking it's reasonable that I couldn't affect anything I have "future" information on, but involving multiple actors either locks causality down such that you might as well have real-world causality (since any lightcone skipping might mess with a sequence of information known to someone else), or that the universe be treated as a nigh-sentient force keeping everything lined up (https://en.wikipedia.org/wiki/Novikov_self-consistency_principle).

Honestly, if I were making a setting, I'd go with it being common knowledge that the universe will arrange coincidences to ensure that time is self-consistent, growing more extreme to match the potential damage to the timestream. You could trick out time by swapping your girlfriend's body in the plummeting car with a nonsentient cloned replacement, but leaving an empty car means that increasingly odd events conspire to have someone who looks very much like her sent back. (Usually at ever increasing frustration to the time traveler, because that's how stories work.)

If we we're talking real life, the multiple actor issue quickly makes it nigh impossible that someone, somewhere wouldn't see a causal break if you made a FTL hop, and indeed the energy costs for a given hop would reveal a lot about who in the universe has information about what other parts. You complicate a ton of things, and have no real-life reason to assume it might work this way.

NichG
2018-11-26, 07:21 PM
Question for the high level physics people.

So, all the lightcone diagrams I've seen are flat. But any number of FTL methods in fiction "Pinch", "Warp" or "wormhole" from one point in spacetime to another.

Assuming such an FTL, able to travel outside it's lightcone, but assume power requirements go asymptotic as it approaches a causality violation with anything that was in it's pre-jump lightcone.

So you detect a ship jumping in, telling you there's a disaster where they jumped from, two years from now. You can jump back there, with relief supplies at any time up to the moment the messanger ship left, two years AFTER it arrived, (though the power requirement goes up the closer you cut it) but no sooner, even though in a hypothetical universal frame, it hasnt happened yet. If another ship arrives or leaves, it resets the earliest arrival time, and if you wait long enough for the light of the disaster to arrive, you cannot violate what you could theoretically see happen.

Does this thoought expiriment hold up?

There are papers which solve for clean systems (billiard-like setups) in the presence of time-travel boundary conditions, and basically it's generally possible to find solutions even in cases where you set things up so that only causality-violating solutions are capable of being self-consistent, even though finding those solutions is (in principle) computationally NP. So causality violations aren't an immediate non-starter mathematically.

The bigger issue for me is, if you take basic linear models from physics such as the wave equation and solve them in the presence of time travel boundary conditions, you find that it forces the solution space into the set of exponentially growing and shrinking periodic functions. So it seems like in the presence of time travel, you would actually need a non-local conservation law in order to retain things such as energy conservation (and if you don't have this, then the universe either exponentially-quickly fills with energy or goes dark).

Lvl 2 Expert
2018-11-27, 10:34 AM
And then there's the quantum entanglement solution, which makes something travel faster than light by making it actually not a thing at all.

Confused? Well, so am I and I'm sort of hoping anyone here can explain it better than I can.

Two particles can when they are in each other's vicinity be entangled. That is, the spin of these particles (which is nothing like what you're currently imagining, but keep imagining it like that anyway because it's a decent model for thinking about it) is now the same, either left or right. That spin can still change, but at the exact moment the spin on one end changes the spin on the other end will change as well. This has been done experimentally with the particles having been transported kilometers apart, researchers are unable to find any delay in the effect. It's instantaneous or at the very least much faster than light. (Or there is something really weird going on that makes us think we're seeing this effect, but assuming physics is to a certain degree right it's instantaneous or much faster than light.)

Here's the catch: You can't tell by measuring the spin of a particle whether it's been entangled. You can only tell two particles have been entangled by comparing measurements from both sides. So you need information send through other channels to know anything happened at all. The effect transfers a single bit of data (was couples or was not coupled), but only does so when combined with a regular light speed or slower channel of information. To further complicate it measuring or altering the spin of either particle breaks the entanglement. (No, I don't know how they even confirmed this effect exists if you per definition can only ever measure the spin on one of these particles. My guess is it has to do with measuring as close after each other as possible and applying statistics about how quickly the spin on the other end would be expected to change after the entanglement broke.)

The creepy thing to me (but I'm not very good at this physics stuff, my world exists between roughly 0.1nm and 40,000km) is that this is the only thing in nature I know off that seems almost designed. Sure, the universal constants are just right, but I can come up with explanations for that, even if it's just "there's a multiverse and most of the universes don't look like ours at all". Life? The explanation without a designer seems easily more fitting than the one with. But quantum entanglement is like something a particularly clever sci-fi writer made up to break his own rules without breaking them. (And then in the third act he uses it to plain break his own rules anyway and the story stops being fun.) Why have a universe that has a speed limit that can be ignored by one thing and one thing only, but it's okay because that one thing can't be used to send messages to the past because it can't be used to send messages? Is there some actual way for the universe to tell which phenomena carry information, and is it the information itself that is stuck behind light speed rather than any other property of matter? It's weird. Completely without consequences for being able to travel or communicate either faster than light or back in time, but weird.

NichG
2018-11-27, 12:28 PM
And then there's the quantum entanglement solution, which makes something travel faster than light by making it actually not a thing at all.

Confused? Well, so am I and I'm sort of hoping anyone here can explain it better than I can.

Two particles can when they are in each other's vicinity be entangled. That is, the spin of these particles (which is nothing like what you're currently imagining, but keep imagining it like that anyway because it's a decent model for thinking about it) is now the same, either left or right. That spin can still change, but at the exact moment the spin on one end changes the spin on the other end will change as well. This has been done experimentally with the particles having been transported kilometers apart, researchers are unable to find any delay in the effect. It's instantaneous or at the very least much faster than light. (Or there is something really weird going on that makes us think we're seeing this effect, but assuming physics is to a certain degree right it's instantaneous or much faster than light.)

That's not quite how entanglement works. The key point is, you aren't ever actually changing the spin, you're deciding what basis in which the spin should have a defined value.

Lets say we have a block of marble that we could cut in various ways. Now, if we cut the marble in a known way such that it is split in two, box the two pieces without looking at them, and send them off in different directions, then we could say that 'the moment someone opens the box, they instantly know the shape of the marble piece in the other box' - but no information was transmitted faster than light here, it's purely virtual (and classical).

The quantum version of that says, you can cut the marble in such a way that you can only ask one question about the shape of the cut on your piece when you open the box - and, having done so, no other information can be obtained (and furthermore, rather than this being a destructive measurement, no other information ever existed about the cut in the first place). Now, you have an interesting situation, because you might ask a different question than the person with the other half of the marble block asks, but there is a global constraint that the information you both obtain about the block together through your questions is bounded in a particular way.

The resolution of this paradox is, neither of your measurements is actually deterministic, so you can't actually know whether you were the one who got the 'real' information while the other guy got noise, or vice versa. Furthermore, even if you compare notes, you can't say that one person in the end had the deterministic measurement and the other had the random one, because that would in some sense establish a privileged order, which doesn't actually exist. So you can at best say that, if you both asked the same question, you will have both gotten the same answer (because it has to be consistent with the original shape of the block that it came from), whereas if you asked different questions then in the end, you both got equally random answers that didn't have anything to do with the initial cut.

That statistical effect, where the distribution of outcomes depends on whether the question asked at both ends was the same, is the 'spooky action at a distance' of entanglement, but because you cannot know whether you asked the same question or a different one without receiving a (slower than light) signal from the other person, or without deterministically deciding to ask the same question ahead of time, you can't use the effect to transmit information.

In the case of spins, the 'marble block' is the fact that both spins must add up to zero, but they could add up to zero in different ways: (1/2, 0, 0) + (-1/2, 0, 0) ; (0, 1/2, 0) + (0, -1/2, 0) ; (0, 0, 1/2) + (0,0,-1/2), for example. The question you get to ask is, in essence, picking an axis (after which, you will always get an answer of +1/2 or -1/2 on that axis, never 0). If you and the other person pick different axes, then (effectively) what happened in the end was that each of you had a 0 that resolved randomly to either +1/2 or -1/2. If you and the other person pick the same axis, then one of you gets +1/2 (and, given that the other person picked that axis as well, you know that the other person has -1/2) and vice versa.

Radar
2018-11-29, 05:33 PM
The thing with entanglement is not the only sort of exception - any quantum measurement is in fact global in a way (at least in the most basic forms of quantum mechanics - there are some local definitions, but I failed to grasp them fully during my student years). How it can be so, You may ask?

Consider the reknown electron self-interference experiment, where it goes through two slits at the same time. So the electron is bahaving like a wave spreading over a considerable area until it is observed, right? It suffices to say that until we detect the electron its wavefunction is reaching the whole detector, but then in a split moment, the electron concentrates itself at the exact spot, where it is detected.

We can do it even more clearly: we can shine light on one of the slits, where electron should pass. The light can scatter from the electron and as such we can detect it. Simple? Pretty much, but consder this: if we see the electron on one slit we also gain information that it most certainly is not going through the other (we do not even need to look there) and as a consequence we see no interference pattern. So a measuremet in one place changes the possible behaviour of an object spanning over an arbitrarily large area.

It is exactly the same kind of problem as with entanglement of many particles, since we have initial information that there is exactly one electron and there are two roads it can take, so the states of the slits (electron or no electron) are entangled.

This also neaty shows that we can entangle objects that aren't close to each other - there aer some neat experiments, where people entagle particles that could never directly interact, which is really cool, if you think about it. But that is another story.

Rockphed
2018-11-29, 06:48 PM
Consider the reknown electron self-interference experiment, where it goes through two slits at the same time. So the electron is bahaving like a wave spreading over a considerable area until it is observed, right? It suffices to say that until we detect the electron its wavefunction is reaching the whole detector, but then in a split moment, the electron concentrates itself at the exact spot, where it is detected.

We can do it even more clearly: we can shine light on one of the slits, where electron should pass. The light can scatter from the electron and as such we can detect it. Simple? Pretty much, but consder this: if we see the electron on one slit we also gain information that it most certainly is not going through the other (we do not even need to look there) and as a consequence we see no interference pattern. So a measuremet in one place changes the possible behaviour of an object spanning over an arbitrarily large area.

It is exactly the same kind of problem as with entanglement of many particles, since we have initial information that there is exactly one electron and there are two roads it can take, so the states of the slits (electron or no electron) are entangled.

This also neaty shows that we can entangle objects that aren't close to each other - there aer some neat experiments, where people entagle particles that could never directly interact, which is really cool, if you think about it. But that is another story.

This is where the "you cannot know both an object's position and momentum" comes from. Technically, the product of the errors of your measurement of position and momentum (which can be extrapolated from the self-interference pattern) is always greater than some number (I think it is h or h-bar, but cannot remember their names). So if you have 0 error on one of your terms, you must have infinite error on the other.

Delusion
2018-12-01, 08:19 AM
This is where the "you cannot know both an object's position and momentum" comes from. Technically, the product of the errors of your measurement of position and momentum (which can be extrapolated from the self-interference pattern) is always greater than some number (I think it is h or h-bar, but cannot remember their names). So if you have 0 error on one of your terms, you must have infinite error on the other.

Planck constant and reduced Planck constant. And IIRC its always greater than h-bar/2.

Phhase
2018-12-02, 10:15 PM
This, by itself, is why it violated causality. How exactly does that violate causality? The way we gather information on Earth is by measuring the light that faraway bodies emit. Of course, since light has a speed, this method breaks down in temporal accuracy over great distances. But the truth, and reality exist without our observation. Out there, the stars are the way they are right now, the only reason we can't know how that is, is because we're interpreting from a material with a travel time. It doesn't seem to me like any great feat of time travel to be at a location that cannot be observed with conventional means, using theoretical means to bridge the gap of distance. ...right? Am I missing something? Completely insane?

georgie_leech
2018-12-03, 12:48 AM
How exactly does that violate causality? The way we gather information on Earth is by measuring the light that faraway bodies emit. Of course, since light has a speed, this method breaks down in temporal accuracy over great distances. But the truth, and reality exist without our observation. Out there, the stars are the way they are right now, the only reason we can't know how that is, is because we're interpreting from a material with a travel time. It doesn't seem to me like any great feat of time travel to be at a location that cannot be observed with conventional means, using theoretical means to bridge the gap of distance. ...right? Am I missing something? Completely insane?

Different reference frames disagree on when "now" is relative to the star. Our reference frame isn't more true than that of one going at near light-speeds, for instance.

NichG
2018-12-03, 01:57 AM
How exactly does that violate causality? The way we gather information on Earth is by measuring the light that faraway bodies emit. Of course, since light has a speed, this method breaks down in temporal accuracy over great distances. But the truth, and reality exist without our observation. Out there, the stars are the way they are right now, the only reason we can't know how that is, is because we're interpreting from a material with a travel time. It doesn't seem to me like any great feat of time travel to be at a location that cannot be observed with conventional means, using theoretical means to bridge the gap of distance. ...right? Am I missing something? Completely insane?

You're missing that the delay depends on how fast you're going as the receiver relative to the emitter, not just the distance from emitter. Therefore, by changing your velocity but remaining in place, you and a third relay can conspire to make the emitter receive a signal before it sends it.

Classically, the delay only would depend on distance in space, not distance in velocity.

Frozen_Feet
2018-12-03, 04:19 AM
@Phhase: you are doing the same error of begging the question as many before you: by imagining "right now" of distant objects, you're implicitly assuming an universal reference frame where time passes at a set speed.

It doesn't. There's no such frame, "right now" is relative and true simultaneity between distant objects isn't a thing. If you introduce a no-delay mechanism between two (or more) distant reference frames, that will work out to negative delay in some third (or fourth etc.) reference frame.

keybounce
2018-12-06, 03:22 PM
Lets make sure we are talking about the same thing here.

My understanding: When 1905 came around, things like the Michelson-Moorly experiment were history and "what does this mean???". The Lorenz contraction formula was known, took pages of calculus to derive, and implied the strangeness of physics that as things moved, they actually changed in shape.

Einstein decided to look at the three "obviously true" beliefs, and asked "what if we get rid of the one that seems to make the most trouble?".

So, he asked, "What if we just make these assumptions:

In an inertial reference frame:
1. The speed of light in a vacuum appears constant (the same) to all observers
2. All laws of physics appear the same to all observers.

From that, he can derive the same Lorenz contraction formula with half a page of high-school algebra, using the same setup that explains the doppler shift.

Without needing to assume that objects change in shape as they move; it suddenly becomes different observers disagreeing on the timing of things.

So, over the next 10 years, he started to ask, "What if, instead of an inertial frame of reference that requires deep space and no gravity, we use a different system of math that can describe what happens with gravity, or a rotating planet?".

That's all G.R. is. The assumption that the speed of light in a vacuum seems to be the same to everyone, as do the laws of physics.

And the conclusions? That you cannot go faster than light while remaining in the universe because you are always ... who was the ancient greek figure that was always trying to play catch-up with the turtle that started ahead of him?

---

When I started, I said that I knew that if you can make two FTL jumps with different frames of reference, the differences in the light cones that represent the frames of reference permit going back in time. I asked about "what if the universe has a special frame of reference such that any FTL jump you could make must be in this one special frame".

We know that G.R. is incomplete.
We know that it disagrees with Q.M. at the extreme.
We know that G.R. cannot describe gravity in the extremes (black holes, big bang).

Just as Newton knew that his rules did not describe Mercury, and there must be something else, so we know that G.R. is only "mostly accurate".

Now, replying to some people here:


General relativity is based on (and named after) the general principle of relativity; that the laws of physics are the same in all frames of reference.

A special frame of reference such as you describe would necessarily vary on those laws of physics which make it special; therefore physical laws would not be the same in all frames of reference, therefore general relativity would not hold.

Why does a privileged frame of reference require that different observers see different laws of physics?

"Everyone agrees that FTL can only be done under special circumstances. Everyone can tell that there's only one set of rules for an FTL jump. And even though different people are moving at different rates, and see a different difference between this special frame and their own, everyone can still determine that there is only one such frame, and can figure out how to calculate the difference between their frame and this one frame".



FTL is impossible because every observer measures the speed of light in a vacuum to be the same. FTL would not just break our understanding of the universe; it would break the universe. Most people do not realize how important the fundamental constants are and how closely coupled to each other they are. FTL would mean nothing is possible any more.

If I can go faster than light inside the universe -- if I can actually outpace a light beam -- then yea, time goes wonky if G.R. remains true. It's moving in the wrong direction, so time must have its sign flipped.

If I can make a jump from A to B and arrive before light, this is a different statement. Doesn't change the observation of light's speed.


I can define a reference frame in which the solar system is not moving or rotating.
I can define a reference frame in which the Milky Way is not moving or rotating.
I can define a reference frame in which the Local Group containing the Milky Way is not moving or rotating.
I can define one in which the Laniakea Supercluster (which includes the Local Group) is not moving or rotating.
I can define one in which the Centaurus Great Wall (which includes the Laniakea Supercluster) is not moving or rotating.
I can even define one in which the center of all currently known extra-galactic structures is not moving or rotating.

But I cannot define "The universe itself" and have any idea what that frame of reference is.

There is no single reference frame that is "The universe itself".

Turn it around. The assumptions of general relativity say that if you define a frame of reference, for instance "The universe itself", then I can define another one that moves relative to that one, and which is just as definitive as that one.

Your frame of reference isn't privileged. It just isn't. It's as arbitrary as any other. If an object is moving at 0.5c in your reference frame, then whatever appears motionless to you is traveling at 0.5c to that reference frame.

The same equations needed to refer to another reference frame from yours will work to refer to yours from that one.

Two things here:

1. Yes, if I define one frame, and call it "special", you can define a different one, and call that "special". But these are just arbitrary definitions by people. This has nothing to do with whether or not the universe itself has a special frame.

2. Just because you (or I) cannot define "the universe itself" and know what that means does not mean that it cannot exist.

Just because we cannot determine "the universe" does not mean that "the universe" does not exist.

jayem
2018-12-06, 04:17 PM
Lets make sure we are talking about the same thing here.
When I started, I said that I knew that if you can make two FTL jumps with different frames of reference, the differences in the light cones that represent the frames of reference permit going back in time. I asked about "what if the universe has a special frame of reference such that any FTL jump you could make must be in this one special frame".

I'm sure it didn't say that when I started, must be some relativistic effect :)

My understanding (which doesn't go to G.R)

If S.R (& G.R) still mostly holds apart from that one special portal-frame, then there will inevitably be some frames where "backwards time-travel" will seem to have occurred. As such there probably will be some weird effects. In a sense that is O.K as these frames just have to accept they are 'in the wrong', but there must be some interesting consequences, especially if/as they can interact with the portal.

If S.R doesn't generally hold, then there are enough real world aspects that cause problems. Do we need to bring in an aether frame? What does the observable universe look like after we've not calculated everything wrongly? What mechanism do we bring in for Muon decay? How does an atom bomb work?

What I don't know, in the derivation it's assumed that the relativistic correction can be separated nicely and only modifies the one bit of the Gallileon transform (I think I have seen it justified). If that is not the case there may be a space for a more complex transformation. I'm pretty sure that will be ugly though.

NichG
2018-12-06, 07:51 PM
If there's a preferred reference frame, you don't have conservation of momentum or energy, and as a result there would be a host of knock-on effects. Elliptical orbits would precess at a different rate than GR predicts. Space would be opaque over a characteristic distance. Etc.

You can of course make theories with spot fixes to all of these issues, but that generally means that if you have to do that, there's something wrong.

Radar
2018-12-06, 08:56 PM
When I started, I said that I knew that if you can make two FTL jumps with different frames of reference, the differences in the light cones that represent the frames of reference permit going back in time. I asked about "what if the universe has a special frame of reference such that any FTL jump you could make must be in this one special frame".

We know that G.R. is incomplete.
We know that it disagrees with Q.M. at the extreme.
We know that G.R. cannot describe gravity in the extremes (black holes, big bang).

Just as Newton knew that his rules did not describe Mercury, and there must be something else, so we know that G.R. is only "mostly accurate".
Well, we do hope it is only mostly accurate, but so far we had no luck in finding any inconsistencies in it. With the birth of gravity wave detection I expect we will gather data that might give a new insight into it. We live in amazing times for science - that one is sure.

It's not that GR disagrees with QM - it is more that they describe different aspects of reality using vastly different language, so it was a big challenge to combine them. I used the past tense, since there are some good candidates for quantum gravity theories and they already bring results - one of the last papers by Hawking made a big impact, since he was able to prove that black holes do not destroy information.

Also, black holes turned out to be extremely easy to describe. In fact, they might be the simplest possible macroscopic objects. :smallwink:

You probably were thinking about the singularity in the black hole and at the beginning of time. I am pretty sure that such extreme and impossible to see objects are outside our current knowledge, but it does not stop people from trying to resolve them. There are some neat cyclic cosmological models which do deal with it more often then not using some version of quantum gravity.


Anyway, as for what would happen, if there was a preferred frame of reference. Short answer? We would already detect it. The thing is, if there is a special frame of reference, where laws of physic look differently, then all the other ones will be defined by how far off they are from the ideal. A good example are non-inertial reference frames - for each one of them we can strictly derive an intertial reference frame. Such asymmetry would have even more unusual consequences, since each symmetry of our reality is directly connected to some conserved quantity (Noether's theorem). It is such a fundamental thing that elementary particle models are build strictly in the language of algebraic symmetry groups. So by cutting out the invariance of law of physics to velocity of a frame of reference, you would be for example breaking conservation of spin along with it. I'm not sure, if there is a subtle way of doing this that would avoid detection frankly.

I was about to say that space-time tunnels don't have to violate causality and allow FTL travel at the same time, but it is not nearly as simple. First, the only causality-consistent way to build one would be to make both ends in one place and then tow one of them to the designated endpoint using regular means. Second, while a single space-time tunnel might not immediately make time travel possible (time dillation on one of the ends would have to accumulate to a larger time-shift then the spatial distance between the ends through regular travel) a network of such tunnels would be easily exploitable and create a time travel possibility very quickly.


If there's a preferred reference frame, you don't have conservation of momentum or energy, and as a result there would be a host of knock-on effects. Elliptical orbits would precess at a different rate than GR predicts. Space would be opaque over a characteristic distance. Etc.

You can of course make theories with spot fixes to all of these issues, but that generally means that if you have to do that, there's something wrong.
For FTL not to cause time travel one only needs to have a distinct velocity of a reference frame or in other words a unique time axis. As such, momentum and energy would be conserved, since the symmetry of translation in space and time would not be broken, but think of the poor spin...

keybounce
2018-12-06, 11:16 PM
I'm sure it didn't say that when I started, must be some relativistic effect :)

I just double checked, in case I had messed up.

My question, in the context of both sci-fi stories as well as real world physics:

But why does FTL have to use the originator's frame of reference? Why wouldn't it use it's own FoR that is based on the FTL medium / whatever you travel through to make the trip?

And once you have a single constant FoR for all trips, doesn't that make time travel by FTL travel impossible?


So ... if the idea is to allow scientific fiction that can have high speed travel and some sense of a "global" clock (Star Trek?), can you say that your FTL system has some special reference frame that applies to FTL so that you don't care what light cone you saw in normal space before you turned on your FTL drive, all FTL drives operate under a special reference frame, and does that prevent the time travel issues normally associated with FTL?

Rockphed
2018-12-07, 03:35 AM
If there's a preferred reference frame, you don't have conservation of momentum or energy, and as a result there would be a host of knock-on effects. Elliptical orbits would precess at a different rate than GR predicts. Space would be opaque over a characteristic distance. Etc.

You can of course make theories with spot fixes to all of these issues, but that generally means that if you have to do that, there's something wrong.

What do you mean by "opaque"? Because for some definitions the universe is opaque if you go far enough away. We are fairly certain that the opacity is related to conditions right after the big bang, but we could be wrong about that. Highly unlikely though.

Radar
2018-12-07, 04:45 AM
I just double checked, in case I had messed up.

My question, in the context of both sci-fi stories as well as real world physics:

But why does FTL have to use the originator's frame of reference? Why wouldn't it use it's own FoR that is based on the FTL medium / whatever you travel through to make the trip?

And once you have a single constant FoR for all trips, doesn't that make time travel by FTL travel impossible?


So ... if the idea is to allow scientific fiction that can have high speed travel and some sense of a "global" clock (Star Trek?), can you say that your FTL system has some special reference frame that applies to FTL so that you don't care what light cone you saw in normal space before you turned on your FTL drive, all FTL drives operate under a special reference frame, and does that prevent the time travel issues normally associated with FTL?

In essence yes, having a single universal frame of reference for FTL would prevent time travel, but it would generally have a lot of other consequences. For the purpose of an SF story it can be dealt with, but for the real world it seems we would notice.

NichG
2018-12-07, 05:47 AM
What do you mean by "opaque"? Because for some definitions the universe is opaque if you go far enough away. We are fairly certain that the opacity is related to conditions right after the big bang, but we could be wrong about that. Highly unlikely though.

For example, in active fluids (in which there is a particular favored velocity), fluctuations in momentum decay exponentially as they travel through the fluid (essentially becoming reset with respect to that special reference frame). So if you're far enough away from something which is injecting a momentum perturbation into the field, you can't detect it. Photons are basically objects with energy and momentum but no rest mass, so one would find that as they propagate through space photons would exponentially lose energy and momentum with distance traveled.

Space isn't totally transparent in the standard model, but it's non-transparency takes the form of photons participating in pair production and scattering off the vacuum. So energy and momentum aren't lost, they're just redistributed over a wider angle. So I suppose that's more like space being blurry than opaque.

Rockphed
2018-12-07, 06:01 AM
For example, in active fluids (in which there is a particular favored velocity), fluctuations in momentum decay exponentially as they travel through the fluid (essentially becoming reset with respect to that special reference frame). So if you're far enough away from something which is injecting a momentum perturbation into the field, you can't detect it. Photons are basically objects with energy and momentum but no rest mass, so one would find that as they propagate through space photons would exponentially lose energy and momentum with distance traveled.

Space isn't totally transparent in the standard model, but it's non-transparency takes the form of photons participating in pair production and scattering off the vacuum. So energy and momentum aren't lost, they're just redistributed over a wider angle. So I suppose that's more like space being blurry than opaque.

Thank you for clarifying.

jayem
2018-12-07, 01:57 PM
[much delayed as didn't click send]


I just double checked, in case I had messed up.

I had, by forgetting the OP by the time I'd read the thread.



My question, in the context of both sci-fi stories as well as real world physics:

But why does FTL have to use the originator's frame of reference? Why wouldn't it use it's own FoR that is based on the FTL medium / whatever you travel through to make the trip?

And once you have a single constant FoR for all trips, doesn't that make time travel by FTL travel impossible?


So ... if the idea is to allow scientific fiction that can have high speed travel and some sense of a "global" clock (Star Trek?), can you say that your FTL system has some special reference frame that applies to FTL so that you don't care what light cone you saw in normal space before you turned on your FTL drive, all FTL drives operate under a special reference frame, and does that prevent the time travel issues normally associated with FTL?

It would make it impossible in that special frame. However the other frames still exist.
NB Wormholes would be (individually) in one or two frames and have been discussed in Hawking's books.

If the lorentz stuff is correct then: from the point of view of a photon going between the two wormholes (from our perspective). It is a stationary electric field and the two wormholes are coming towards it at c, and are basically contracted to be adjacent. The universe would contain the now-effects of the ship coming out the wormhole and about to go in.
Obviously for all of us close to the wormholes frame of reference differences will be tiny, but I think once we interact with people in non-wormhole frame because they now have reversed causality in their frame, by them exploiting us and the wormholes. In turn there may be some way to exploit this (though maybe only to getting FTL messages in other directions).

If not then we need to explain magnetism.

Douglas
2018-12-07, 03:12 PM
It would make it impossible in that special frame. However the other frames still exist.
NB Wormholes would be (individually) in one or two frames and have been discussed in Hawking's books.
I'm pretty sure that, so long as the special frame itself isn't moving FTL, no matter what other frame you use it can't send you farther back in time than how long it would take to travel back to your starting point at light speed in the normal frame. Even if you can instantaneously teleport arbitrary distances in the special frame, you can't make circular causality by teleporting one way and then flying back non-FTL - and teleporting for both segments of the trip just sends you forward in time on the return trip, still arriving after you left. You can't make a causal loop using this method of FTL.

This kind of FTL does break local conservation laws, though. A traveler spontaneously appears somewhere, and the amount of mass in the universe increases. Later, but not too much later, that same traveler departs into the past (of a distant place), and the change in mass has globally balanced out.

To make circular causality, you need to travel FTL in at least two different frames. One-frame FTL lets you travel back in time in some reference frames, but never by enough to make it possible to close the loop.

Lvl 2 Expert
2018-12-17, 07:15 AM
That's not quite how entanglement works. The key point is, you aren't ever actually changing the spin, you're deciding what basis in which the spin should have a defined value.

It took me a while to let this sink in, but I've found a metaphor that works for me. You'rs basically saying that the spins are not coupled, they're synchronized. They will be making the same/the exact opposite/something like that motions (if you can call it that at this level) because they're been set to the same/the opposite/complementary starting conditions. By measuring you disrupt this pattern, so the spin here no longer says anything about the spin back there. But even when it did say something about the spin back there, it can't say anything about anything that interacted with the spin back there, because it only tells me they were synchronized at some point in the past.

That would be quite an elegant solution. I'm kind of hoping they prove that one. (Also kind of not, but that's the trekkie in me talking.)

NichG
2018-12-17, 07:50 AM
It took me a while to let this sink in, but I've found a metaphor that works for me. You'rs basically saying that the spins are not coupled, they're synchronized. They will be making the same/the exact opposite/something like that motions (if you can call it that at this level) because they're been set to the same/the opposite/complementary starting conditions. By measuring you disrupt this pattern, so the spin here no longer says anything about the spin back there. But even when it did say something about the spin back there, it can't say anything about anything that interacted with the spin back there, because it only tells me they were synchronized at some point in the past.

That would be quite an elegant solution. I'm kind of hoping they prove that one. (Also kind of not, but that's the trekkie in me talking.)

That describes the classical version of entanglement. The quantum version is a bit more complex, in that the spins are described as having the properties (x,y,z) such that this synchronization would exist for any one of them. However, those axes aren't independent - there's a relationship such that you can't simultaneously know x, y, and z (much like position and momentum) because they aren't actually independent degrees of freedom in reality but are related by a constraint of constant angular momentum. Or, alternately, you could look at polarization of a photon which basically has two bases that are both equivalent to describe it (you can think of it as one basis aligned with the x,y axis, and another at a 45 degree angle to that).

If you measure x and someone else measures y for example, then the measurements have nothing to do with one-another. However, if both people were to measure x or both people were to measure y, they would see that synchronization is respected along the axis they both chose to measure. The statistics of that are subtly different than the classical statistics - specifically it exists in the correlation between the measurements rather than any measurement on its own. In order to evaluate the correlation, you have to bring the results of both measurements to the same place in space-time, which is why despite there being a deviation from the classical result it does not imply that you can transmit information faster than light.

The specific statistical quantity is given by Bell's Inequality.

Radar
2018-12-17, 12:17 PM
It took me a while to let this sink in, but I've found a metaphor that works for me. You'rs basically saying that the spins are not coupled, they're synchronized. They will be making the same/the exact opposite/something like that motions (if you can call it that at this level) because they're been set to the same/the opposite/complementary starting conditions. By measuring you disrupt this pattern, so the spin here no longer says anything about the spin back there. But even when it did say something about the spin back there, it can't say anything about anything that interacted with the spin back there, because it only tells me they were synchronized at some point in the past.
And to even know that there was a synchronisation, you need to examine many experiments and have resuts from both particles.


That would be quite an elegant solution. I'm kind of hoping they prove that one. (Also kind of not, but that's the trekkie in me talking.)
They actually did as long as you assume that laws of physics have to be local. Aside from using standard two-photon entanglement (better then for particles, since it allows us to avoid any possibility of interaction after creation), they did it with three-photon entanglement, for which differences between quantum (assumption that the result of an experiment is truly random within the given constrains) and hidden variable (assumption that the results of an experiment is predetermined) theory are much more drastic.

For more information see Greenberger–Horne–Zeilinger entanglement. They are doing really cool experiments on this and quantum computers.

Lvl 2 Expert
2018-12-17, 01:56 PM
And to even know that there was a synchronisation, you need to examine many experiments and have resuts from both particles.

Yeah, but I knew that part. That was what I figured was so spooky about the situation. Information travels faster than the speed of light, but without a second slower than light channel it can only be useless information. That's not a natural law, that's a made up rule.

It's almost a shame that what's actually happening is kind of normal.

(Also: Thank you both for the extra info.)